Download as pdf or txt
Download as pdf or txt
You are on page 1of 36

LSAT

PrepTest 3
Explained
a guide to the december, 1991 exam
 2006 Kaplan, Inc.

All rights reserved. No part of this book may be reproduced in any form, by photostat, microfilm, xerography, or any other
means, or incorporated into any information retrieval system, electronic or mechanical, without the written permission of
Kaplan, Inc. LSAT is a registered trademark of the Law School Admission Council.
Section I: Logic Games

SECTION I: Key Deductions: and we have “NO R B” under Nat to


indicate that he wants something different than his
LOGIC GAMES date, Olive. Our work with combining Rules 2 and 4 will
prove quite useful. We should also include Rule 1 in
Game 1: Three Couples At Dinner our considerations.
Since Olive, Nat’s better half, orders roast beef (Rule
Questions 1–7 5), Nat can’t, and Nat’s choices were limited to begin
with (Rule 4). Therefore, Nat must order either pork
The Action: We can tell from the first three sentences chops or veal cutlet. We can also deduce that Lewis
that the first game is a matching game; we’re given 6 won’t order swordfish since he won’t duplicate Marie’s
people (3 couples—John and Kate, Lewis and Marie, choice. Notice that while we have definite choices for
and Nat and Olive), and are asked to match each to two out of the three women, we also have lots of
one par ticular entree (pork chops, roast beef, information on the men, especially Nat. Once you get
swordfish, tilefish, or veal cutlet). Notice that with 5 the setup to this point, you can bet that more than a
entrees and 6 people, there has to be some couple of answers will spring from John and Nat running
duplication of entrees. The Key Issues are basic: out of entrees to order thanks to Rules 1 and 2.
1) Who orders what entree?
The Final Visualization: Here’s what we have before
2) Who can, must, or cannot order the same entree as moving on to the questions:
whom?
P/R/V No S P or V
The Initial Setup: To sketch the information, simply PRSTV J=K L=M N=O
write J, K, L, M, N, and O across the top of the page
with double lines between K and L, and M and N, in MEN NOT SAME S R
J=L=N
order to visually break the 6 people into their
respective couples. If you wish, you can also
distinguish between men and women, by writing tone The Big Picture:
group in capital letters, and the other in lowercase. Add • If rules are given to you in the negative, turn them
in the list of the entrees off to the side, and you should around to the positive. The fact that John and Nat
have something like this to begin with: don’t order swordfish or tilefish is not nearly as
useful as deducing that they MUST order one of
PRSTV JK LM NO the remaining entrees; pork chops, roast beef, or
veal cutlet.
The Rules: • Always think actively. Flagging potential major
rules and thinking through possibilities are just
Start with the two most concrete rules, Rules 3 and 5. two techniques that can really save time and aid
3) An “S” under Marie will remind us that she opted for in conquering the lion’s share of the questions.
the swordfish. • Work with the most concrete rules first; anything
5) “R” under Olive means that it’s roast beef for her. that can go into the sketch immediately will help
1) This rule adds a nice touch—it allows the man and to lay the groundwork for the rest of the Final
woman in each couple to share. One easy way to Visualization. But once they’re incorporated
remember this is to write a “≠” between the members (Rules 3, 4, and 5 are now built into the
of each couple. visualization above), turn your attention to the
2) You could write “men never order the same,” but rules that will most likely govern the rest of the
you’re always better off being more specific: J≠L≠N. game, which in this case are Rules 1 and 2.
4) No fish for John or Nat, which means that swordfish
and tilefish are out for them. But there are only five
entrees to begin with, which means that these two
guys are restricted to either pork chops, roast beef, or
the veal. Indicate that somewhere in your sketch.

1
PrepTest 3 Explained

The Questions: 7. (D)


Deal with rule changes first. Since Marie has
1. (D)
swordfish, now so does Lewis. Olive has roast beef;
We deduced that Lewis will not order swordfish since now, so does Nat. Rule 2 is still in effect, so John is
Marie, his date, orders it. However, there’s nothing that left with a choice of pork chops and veal cutlet, which
prevents him from ordering any of the other entrees, so means that Kate is limited to the same. They both can
the other four must be included in the list. opt for the pork chops, so (D) can be true.
(A), (B), (C), and (E) are impossible.
2. (B)
Not much to do but check out each choice. (A) is
knocked out by a combination of Rules 3 and 4. (C)
violates Rule 2. Rules 1 and 5, taken together, signify
that (D) can’t be true, and Rule 1 by itself makes (E)
impossible. That leaves (B); Kate would have to order
either pork chops or veal cutlet, because that’s all Nat
can eat, but that’s eminently feasible.

3. (A)
Here’s our Big Deduction: Nat has to order either pork
chops or veal cutlet, and choice (A) therefore must be
a true statement. (B) and (C) are impossible, while (D)
could be true, but need not be (Lewis could order
tilefish) and (E) merely could be true as well, but would
be false if Kate ordered fish.

4. (E)
John orders the veal, so Nat and Lewis cannot (Rule 2).
This makes pork chops Nat’s only choice. Just for the
record, Kate ordering pork chops and Lewis ordering
roast beef kills choices (A) through (D).

5. (C)
Nat doesn’t order pork chops (no one does), so he
orders veal cutlet. Oops, not one of our choices, so we
must continue. John now must order the roast beef
(now his only choice). Still not an answer choice
(though it does rule out choice (A)). Lewis’ choices are
now limited to tilefish, whatever that may be. That’s a
choice—(C). (B) and (E) could be true, (D) never.

6. (A)
Rule 2 forces Nat to order the veal cutlet, and the
same rule mandates that John settle for the roast beef.
John’s complete and accurate list is simply roast beef,
choice (A).

2
Section I: Logic Games

Game 2: Seven Families in Houses possibilities: M lives in either 3 or 5. Let’s say M


occupies the 5th house. Since the P…L pair must be
Questions 8–13 east of K, we get this:

The Action: A fairly straightforward sequencing game, 1 2 3 4 5 6 7


with a very minor wrinkle: the use of the terms “west” O/N K M P L
and “east” to refer to the different sides of the
sequence. We’re asked to order seven families (K, L, R, O, and N are left to fill in the 1,2 and 3 houses. R,
M, N, O, P, and R) in seven houses lined up west (left) you remember, can’t be on either end, so that family
to east (right) on one side of a street. The Key Issues would be in either 2 or 3 in this situation, leaving O and
are: N for the other spots, in either order.
1) Which families can, must, or cannot occupy which The second scenario is a little more flexible, and arises
house? if we instead place M 3rd. P and L still need to be to
the right of K, and must still maintain that K..P..L
2) Which family can, must, or cannot occupy a house ordering. The other 3—N, O, and R—would fill the
adjacent to what other family? remaining spots, so long as R isn’t first or last. This
The Initial Setup: Keep this setup simple; seven ordering isn’t as determined as the other, and naturally
dashes or the numbers 1 to 7 from left to right on your doesn’t help us as much, but it’s still good to consider
page will suffice. You may wish to jot down a “W” to the the possibilities in advance, which we’ve done.
left and an “E” to the right of the sketch, just for good The Final Visualization: Here’s what we’re armed with
measure, to remind you which side is which. List the to tackle the questions:
roster of families off to the side, and you’re ready to fill
the Rules into this basic setup: M
W no R no R E
W E KLMNOPR 1234567
KLMNOPR 1234567 K
P…L
The Rules: The Big Picture:
2) Once again, the most concrete rule first: The Kahns • When one entity is restricted to two options, it’s
live in the fourth house from the west. Don’t let the usually worth it to try out the possibilities. A rule
awkward wording throw you. This simply means that of thumb for dealing with such options: If you can
the Kahns live in house 4, so put that directly into the use the rules to narrow down the concrete
sketch. possibilities to two, it may be wise to get those
1) “No R” over houses 1 and 7 should help us keep possibilities solidly down on paper. And when
this in mind. you’ve taken the time to do this, remember to
refer back to this work in the process of handling
3) Since the Muirs live next to the Kahns, who live in
the questions.
house 4, the only possible houses for the Muirs are 3
and 5. Place an “M” with arrows pointing to houses 3 • Always strive to build a rule directly into your
and 5 into your sketch. master sketch. Only when that is not possible,
should you rewrite the rule in your own words.
4) Since the Kahns live in house 4, this rule tells us
When rewriting a rule is not feasible, you should
that the Piatts can only live in houses 5, 6, or 7. But
underline or circle it for future reference.
the Piatts can’t live in 7, because the Lowes must live
to the east (right) of them. So the “P…L” chunk will • In such a straightforward game, the testmakers
have to fall somewhere within houses 5, 6, and 7. Jot probably felt that they had the right to include a
that down, and we’re ready to move on. slight variation on the sequence—this west to
east business. But aside from the language, this
Key Deductions: There’s no major deduction here, but really doesn’t change anything. Don’t let a little
there are only a limited number of ways the sequence wrinkle ruin an otherwise simple game for you:
could play out. It’s therefore worth quickly exploring the Take command.

3
PrepTest 3 Explained

The Questions: 12. (A)


Here we can deduce that the second scenario above is
8. (C)
operative: The only way for the Owens to live to the
The Lowes live in house 7 in our first possibility above, east, or right, of the Muirs, is for M to live in 3, with O,
but that’s not a choice. (A), (B), (D), and (E) directly P and L fitting somewhere in 5, 6, and 7. The two
violate Rules 2, 3, 4, and 1, respectively, which leaves remaining entities, N and R, will therefore take houses
only (C). A glance at our sketch would quickly confirm 1 and 2, respectively (R can’t take 1 from Rule 1). The
the same result. choice that corresponds to this is (A): K must be east
of M. (B) is false, and (C) through (E) are possible only.
9. (A)
This one’s also pretty much a gimme. We can easily 13. (D)
infer from Rule 4 that the Piatts live between the Kahns The testmakers liked question 12 so much, here it is
and the Lowes. Therefore, there’s no way that the again (nearly). So we should consult the same sketch
Kahns can live next to the Lowes. The rest of the as the one for number 12: M is in 3, K is in 4, O, P, and
families in (B) through (E) can live next to the Kahns. L are relatively free to float between spaces 5, 6, and
7 (as long as P is before L), N is in house 1, and R is
10. (C) in house 2. Scan down the list for adjacent families; N
The info in the stem puts M in house 3, which conjures and R must live next to one another, specifically in
up the second scenario we discussed earlier. This was houses 1 and 2, respectively. (A), (B), and (E) contain
the less helpful scenario, so let’s check out the pairs that are possible neighbors, but need not be,
choices. whereas the pair in (C) are definitely separated by the
(A) R can live next to both K and P: K in 4, R in 5, P in Rutans.
6, and L in 7, with N and O floating between 1 and 2.
(B) is possible too: M,K,P,R,L in 3 through 7, again
with N and O floating at the beginning.
(C) M is in house 3, K is in 4, and P is either in 5 or 6.
It’s therefore not possible for the Rutans to live
between the Muirs and the Piatts, making (C) our
answer.
(D) and (E) Either O or N can live in house 1, and with
R in 2 and M in 3, so both of these are possible.

11. (A)
N in 3 points us to our first scenario:

1 2 3 4 5 6 7
N K M P L
We’re left with R and O for houses 1 and 2, and Rule
1 forces R into house 2 and O into house 1. The entire
setup of families to houses is complete: O, R, N, K, M,
P, L. The Owens clearly do not live next to the
Newmans. Choice (A) it is.

4
Section I: Logic Games

Game 3: Car Display new…nothing. Be careful, a new car can either be a


research model or a production model. “If U, then P”
Questions 14–19 and “if R, then N” should help us remember this.
3) None of the cars in the exhibition are both research
The Action: This fairly complex game involves matching models and sports cars. Again, be careful when
cars with 3 different characteristics (family or sports considering what this means. If a car is a research
car, new or used, and production models or research model, then it must be a family car. If a car is a sports
models), and then with a fourth characteristic, the floor car, then it must be a production model. If a car is a
they’re displayed on. Important to note that each floor family car or a production model, then we know nothing
contains all or one of each of the three different (or at least nothing new). “If R, then F” and “if S, then
characteristics, i.e. there will not be both new and used P” will aid in sorting it out.
cars on a single floor. There’s only one major Key Issue
that every question will involve: Key Deductions: The results of our work with Rules 2
1) What characteristics are exhibited by the cars on and 3 certainly qualify as “Key Deductions.” However,
each floor? there’s even more to piece together. Rule 2 yielded “if
Research, then New,” and Rule 3 yielded “if Research,
The Initial Setup: Since we’re dealing with 3 floors then Family.” To make it easier we can combine those
each containing 3 characteristics, one possible way to to get “if Research, then New and Family.” Also, don’t
visualize this is to draw a 3 by 3 table. We can label the neglect your grid. Rule 5 resulted in a big “U” under
vertical side of the box 3, 2, and 1, from top to bottom, floor 3, and thanks to Rule 2, we know that “if Used,
and each box across horizontally would represent one then Production.” Therefore, the floor 3 cars must be
of the three characteristics: style, age and type. This production models. Two of the three characteristics
way we can picture each floor and it’s relevant traits: are taken care of for that floor.

The Final Visualization: Here’s what we have just


F/S N/U P/R before facing off with the questions:
3
2 S
If F+S, then F/S N/U P/R

1 If U, then P F 3 U P
If R, then N+F 2
The Rules:
If S, then P 1 N
Where to start? As usual, with the concrete rules.
4) The cars on floor 1 are new. An “N” in the
appropriate grid box will remind us that ALL of the cars The Big Picture:
on floor 1 are new. • Concrete information is much more powerful than
5) The cars on floor 3 are used. A “U” in the the abstract. Always ask yourself, and focus on,
appropriate floor 3 grid box means that all the cars what you do know, instead of what you don’t
exhibited there are used. know.
1) This rule is fairly lengthy, so take your time and be • Remember the contrapositive. Don’t forget the
sure that you fully understand it. IF (that’s a big if) contrapositive. Always recall the contrapositive.
there are both family and sports cars, then all of the Never neglect the contrapositive.
family cars will be on lower floors than any of the • It’s very possible that you would have been best
sports cars. So if there are both kinds, floor 1 must be served leaving this game until last—many test-
family cars and floor 3 must be spor ts cars. takers who took control of the section did just
Understanding this rule is far more important than that.
exactly how you choose to represent it on the page.
2) There are NO cars that are both used and research
models. What does this tell us? If a car is used, then
it must also be a production model. If a car is a
research model, then it must be new. If a car is

5
PrepTest 3 Explained

The Questions: 18. (D)


Rule 3: All research cars are family cars. Stem: All new
14. (A)
cars are research cars. Therefore: All new cars are
If sports cars are on exactly 2 floors, then family cars family cars, which happens to be choice (D).
must be on exactly 1 floor, and Rule 1 applies: Sports
Family cars need not be anything in particular, so
cars must be on floors 2 and 3, which eliminates
choices (A) through (C) are out.
choices (C) and (E), and family cars must be on floor
1, which contradicts choice (B). Floors 2 and 3 contain Similarly, in (E), a production model car could be a
sports cars which are production models (Rule 3). sports car.
Choice (D) is therefore out. Only choice (A), research
models on floor 1, is possible. 19. (A)
If all production models are used, then a car that is not
15. (D) used (new), must not be production (research). So floor
No new information here, so move right on to the 1’s new cars must be of the research variety, which
choices. Rule 2 kills choices (B), (C) and (E), since we means that floor 1’s cars must also be family cars
know that at least one floor has used cars. Rule 3 axes (Rule 3). We get this in choice (A).
choice (A). We’re left with choice (D), and it is indeed (B) through (E) all could be true, but also could be
possible that exactly one floor has research models (as false.
long as that floor is floor 1 since it would have to
contain new research model family cars).

16. (D)
Another non-if, this one a “must be true” with no new
information. A quick scan through the choices in
search of one of our earlier deductions turns up choice
(D)—production models on floor 3. (A), (B), and (C)
are possible only, while (E) is totally impossible.

17. (E)
Since we deduced that floor 3 contains production
models, the 2 floors with research models must be
floors 1 and 2. A quick glance at your scratchwork tells
you that these research cars must be new family cars
(Rules 2 and 3). Notice that this takes care of eight out
of the nine possible boxes in our table. Also notice that
the question is a “can be false” question. Here’s
where the on-the-ball test-taker realizes that it’s a good
bet that the answer to the question will almost
certainly deal with the one box that’s still unresolved,
which deals with the family/sports issue on floor 3
(everything else is determined, and therefore must be
true). Choice (E), family cars on 3, could be false, and
therefore does the trick. Choices (A) through (D)
correspond perfectly to the situation, so naturally they
all must be true.

6
Section I: Logic Games

Game 4: Pilots and Copilots 5) Anna will only fly in plane 1 or plane 4. Since
everyone is flying in the show, we know that one of
Questions 20–24 these planes must be used. To build this into our
master sketch, write “A” with arrows pointing to planes
The Action: In this grouping game, we’re asked to 1 and 4.
distribute six entities—three pilots and three 6) Dave only flies in plane 2 or plane 3—“D” with
copilots—among four planes—planes 1, 2, 3, and 4. arrows to 2 and 3 takes care of this.
While the numbering of the planes may suggest
sequencing, your overview should have dispelled that Key Deductions: Not much in the way of deductions,
misconception right away: There’s no mention that but there are a few issues that are worth working out
these planes are “in a row;” they’re not numbered before hitting the questions. First, the numbers: No
“from left to right;” and none of the rules say anything plane flies without a qualified pilot aboard. But we have
about people being “in adjacent planes,” or anything only three qualified pilots, which means that a
like that. Our job is simply to distribute the pilots and maximum of three of the four planes are flying; at
copilots into the planes. The main Key Issue, least one is going to remain empty and on the ground.
therefore, is a grouping concern: Also, since a plane can’t fly without a pilot (Rule 3), we
1) Who’s in what plane? And by extension: Which know that one of the pilots must join Dave (a copilot) in
pilots and copilots can, must, or cannot fly in the same either plane 2 or plane 3. It can’t be Anna, since she’s
plane as which other pilots and copilots? in plane 1 or 4, so Dave must fly with either B or C, in
plane 2 or 3.
The Initial Setup: Keep this setup simple; four circles
or columns, numbered 1 to 4, can represent the The Final Visualization: Here’s what we’re armed with
planes. Then list the pilots and copilots off to the side: to reel in these five questions:

A
P C 1 2 3 4 D
ABC DEF
At Least 1 2 3 4
1 Pilot

The Rules:
1) and 2) You most likely already used these rules to
get a handle on the entities. However, some test takers Either B or C
overlooked a key element of these rules—that the with D
pilots and copilots “are all aboard planes that are flying The Big Picture:
in the show.” This means that everyone flies. Selecting
who flies isn’t an issue; they’re all up in the air. The • Don’t forget that critical reading is incredibly
only question here is which plane each person is in. As important for Logic Games as well as for the
for listing the entities, some find it helpful to use other sections of the test. Sometimes, the
capital letters for the pilots and lowercase letters for testmakers only imply—i.e., don’t clearly spell
the copilots, some don’t. Do what’s easiest for you. out—information that proves to be vital to the
game. Interrogate the stimulus: “Do all pilots
3) Translation: Every plane that’s flying needs one fly?” Yes. “Do all copilots fly?” Yes. “Do all planes
pilot—at least one. This rule says nothing about fly?” No—in fact that’s impossible. As many
copilots, nor does it imply that only one pilot may fly in planes fly as are needed to get the 6 people aloft.
a particular plane. So far, it’s quite possible that
exactly one pilot flies a plane without a copilot, just as • Attempt to weed out the game’s major concern.
it’s possible for more than one pilot to fly in one plane. Not every game has one specific major concern,
but in the ones that do, focusing on this aspect
4) This rule is basically a loophole closer to ensure will help you in almost every question. In this
that no one from the audience or anywhere else rushes case, our major concern is “who will accompany
out and pilots a plane.

7
PrepTest 3 Explained

Dave?” At least Bob or Cindy—if not both—must 23. (B)


do so, so you should find it helpful to incorporate This is ver y much like a standard acceptability
new information from the question stems into the question. (D) and (E) bite the dust thanks to Rule 6.
context of how it relates to taking care of Dave. Dave can’t fly in 1 but must have a pilot with him in
• Use the hints provided for you in the question plane 2 or 3, so any prospective plane-1 crew that
stems. The phrase in question 23’s stem “If included all three pilots would be impossible. That
plane 1 is used” clearly implies the possibility of eliminates Choice (A). (C) leaves only Anna to join
cases in which it is not used. True, this question Dave which isn’t possible. That leaves us with (B)—
comes towards the end of the game, but very Anna, Bob, Ed, and Fran—a perfectly acceptable crew
often in a game whose action is slightly unusual for plane 1, with Cindy and Dave in plane 2 or 3.
or complex, one of the first few question stems
will help to clarify the situation. 24. (C)
Dave and one pilot can’t be in 4, but everybody else
The Questions: can be. That’s the maximum, four people, choice (C).

20. (B)
Anna is in plane 4 and Dave is in plane 2. Well, we
knew we’d have to focus on the Dave situation, and
here it is right off the bat. Since Bob or Cindy (or both)
needs to accompany Dave, Cindy in 3 forces Bob into
2 with Dave.
(A) No reason why Cindy couldn’t fly in plane 2 with
Dave.
(C) No, Bob could fly in plane 2.
(D) No; if Bob flies in plane 4, Cindy would have to fly
with Dave in plane 2.
(E) If Cindy flies in plane 2 with Dave, then Bob could
fly in any of the planes.

21. (C)
Anna can’t fly with Dave (Rules 5 and 6). If Bob joins
Anna, he can’t fly with Dave. So Cindy would have to fly
with copilot Dave, choice (C). Choices (A), (B), (D),
and (E) all could be true, but none of them must be
true.

22. (D)
Cindy and Fran fly alone, Anna never flies with Dave, so
Bob must fly with Dave, choice (D). (A) is dead wrong,
and (B), (C), and (E) are merely possible.

8
Section II: Logical Reasoning

SECTION II: contraire choice.(C) also seems to be supporting the


work of the investigators, rather than casting doubt
LOGICAL REASONING upon it. If the investigators’ first impressions of people
are confirmed by their later observations of the people,
1. (C) then all this proves is that the investigators are good
In this parallel reasoning question, the stimulus is fairly observers and good judges of character, right? This
easily to symbolize: If you have a lot of money in the certainly doesn’t indicate a flaw in their method.
bank, (if X), your spending power is great (then Y). If (D) is more useless background info. Whether it’s likely
your spending power is great, (if Y), then you are happy that there are more Geminis on the street than in the
(then Z). The conclusion combines the first two general population has little to do with the possible
sentences: If you have a large amount of money in the influence of astrology on personality and in no way
bank, (if X), then you are happy, (then Z). So the criticizes the investigators’ research methods.
complete argument in symbols would read ``If X, then
Y. If Y, then Z. Therefore, if X, then Z.’’ Choice (C) has 3. (D)
the same form as the stimulus argument: If you swim, The conclusion: North American children can be made
(if X), then your heart rate increases (then Y). If Y, then physically fit only if they have daily calisthenics at
you are overexcited (then Z). Therefore, if X, then Z. school. The evidence: European children, who engage
(A) If you have good health, (if X), then you can earn a in calisthenics each day at school, are stronger, faster,
lot, (then Y). If Y, then you can buy an expensive house, and less easily-winded than North American children,
(then Z). Therefore, if X, then you can have a whose schools rarely offer daily calisthenics programs.
comfortable life, (then Q?) The assumption: calisthenics are a key part of the
(B) If you drink too much, (if X), then you will feel sick, European children’s physical fitness. Choice (D) neatly
(then Y). If X, then you will have no money left, (then Z). paraphrases this assumption, and is the correct
Therefore, if Z, then Y. choice.
(D) If you exercise, (if X), then you are fit, (then Y). If X, (A) confuses necessity with sufficiency. The author
then you are exhausted, (then Z.) Therefore, if Y, then Z. assumes that calisthenics are necessary to insure
(E) If you have a lot of money in the bank, (if X), then physical fitness; whether they are sufficient to do so is
you are confident about the future, (then Y). If you are a different question.
optimistic, (if Z), then Y. Therefore, if X, then Z. (B) This need not be so in order for the conclusion to
remain valid; try the Denial Test, and you’ll be
2. (E) convinced.
The stimulus concludes that a person’s birth sign (C) and (E) are beyond the scope of the argument. The
influences their personality. We want to cast doubt on author never mentions health (C) or nutrition (E), so he
the conclusion that the subjects, because they were or she needn’t assume anything about either.
Geminis, were more sociable and outgoing than the
average person. The key here is that the subjects were 4. (B)
all volunteers. Would shy, introverted people offer to If buildings have to be unobtrusive in order to be
appear on a TV program? Probably not. (E) picks up on inviting and functional, and modern architects produce
this, offering a plausible explanation which suggests buildings that are not functional because their strong
that the method itself, and not astrology, could have personalities take over their work, then we can
produced the results. conclude that these specific architects are producing
(A) is an au contraire choice. Since the test results buildings that are not unobtrusive.
matched the investigators’ impressions, the test would (A) and (D), much like (A) in the previous question,
seem to be more valid if it were administered by confuse necessity with sufficiency. It may be necessary
impartial people. for a building to be unobtrusive (A) and take second
(B) is useless background information; since astrologers place to the environment (D) in order for it to be inviting
are far from impartial in this discussion, their claims and functional, but that doesn’t mean either is
cannot be used as plausible evidence. In any case, to sufficient for that to happen.
say that non-Geminis are less sociable than Geminis (C) Simply because an architect has a strong
supports the investigators’ case, so this is also an au personality, it doesn’t mean that he or she must let

9
PrepTest 3 Explained

that personality take over. We know that in some cases economic hardship for users of transportation. The
this has been true, but perhaps some architects can evidence for this conclusion is that if the fare doesn’t
control their strong personalities, and still be able to increase, service will be cut, and a large loss of
produce unobtrusive buildings. ridership will occur. The author doesn’t given any
(E) It’s never stated that an architect can’t put his or reason why the fare hike should occur; she only
her personality into a building without having it be outlines the negative consequences that will result if it
obtrusive; we’re told only that architects who let their doesn’t occur. (C), which states that the author arrives
strong personalities take over their work haven’t at a conclusion indirectly by rejecting an alternative,
produced buildings that are functional for public use. explains her strategy quite clearly.
(A) directly contradicts the stimulus; the author freely
5. (E) admits that some riders will experience hardship
The director argues for the funding of the because of the hike.
megatelescope on the grounds that the whole world (B) The argument does explore the other side of the
benefits from new technology and new inventions, and issue, and its consequences, but there’s no indication
that funding for these ventures is not beneficial to only that a supporter of an alternate position would face a
the scientists themselves. The director uses Maxwell, contradiction.
Newton and Einstein as examples of scientists who (D) The author doesn’t argue by defending her
were not limited by a lack of funding, and were, proposal against objections leveled at the alternative.
therefore, able to make discoveries that benefited the Rather, she herself raises objections against the
whole world. Clearly, the director is drawing an analogy alternative and argues that they are stronger than
between the megatelescope research and the research those that can be raised against her proposal, which is
of those three great scientists. That’s a pretty heady to raise fares.
comparison to make; the author needs to present
(E) is out in left field. There’s no mention of past
evidence showing that the megatelescope research
actions with regard to fare increases, and the author
may approach the same level as that done by these
certainly doesn’t prove anything by using evidence from
great scientists. (E) is therefore the strongest criticism
the past.
of the argument.
(A) The director is using Newton and the others as 7. (A)
examples of earlier scientists who made great
Everyone who participates in local politics has an
discoveries; there is absolutely no appeal to the
influence on the community’s values. Since some of
authority of these long-dead people on the subject of
those people are selfish oppor tunists, we can
the megatelescope. And since they’re the only experts
conclude that some selfish opportunists have an
mentioned, (A) isn’t a possible criticism of the
influence on the community’s values.
argument.
(B), (C), (D), and (E) could be true. None of these
(B) is irrelevant. It really doesn’t matter who opposes
must be true.
the development or funding of the megatelescope,
because the opponents of this argument aren’t being
8. (B)
attacked by the director; only their point of view is
questioned. The discrepancy: lighteners, which are without
cholesterol, raise the blood cholesterol levels of
(C) is a distortion. Charging that someone’s point of
consumers higher than does the milk, which contains 2
view is dangerous is distinctly not the same as
milligrams of cholesterol. The key here is that
launching a personal attack on that person.
lighteners contain more saturated fat than milk. So
(D) makes an irrelevant distinction. The word “benefit” we’re looking for a choice that will explain the
in either the economic or the intellectual sense would relationship between saturated fat and cholesterol,
have the same effect on the argument, since the with regard to blood cholesterol levels. (B) does just
astronomers, along with the rest of the world, could that.
reap either or both kinds of benefits from the funding.
(A) is useless background information. The nutritionists’
recommendation doesn’t explain why a product which
6. (C)
doesn’t contain cholesterol, like a lightener, would
The author argues that a fare hike of forty percent produce more blood cholesterol than a product like
must be implemented even though it will cause milk, which does contain cholesterol.

10
Section II: Logical Reasoning

(C) adds an irrelevant distinction. Light cream has people unhappy. In this case, serious financial
absolutely no bearing on the issue, which is the relative problems are sufficient to make people unhappy, but
effects of lighteners with no cholesterol, and milk with that doesn’t mean that this condition is necessary for
cholesterol. unhappiness.
(D) is irrelevant, because it brings up a lightener that (B) and (C) also confuse necessary and sufficient
doesn’t contain coconut oil. So what if this type of conditions. Notice that when we negate the conditions,
lightener has less fat and cholesterol than milk? It still the paradigm shifts: Not having serious financial
doesn’t help to resolve the discrepancy involving problems is necessary to being happy (according to the
coconut oil lighteners. passage), but it is not sufficient.
(E) This choice explains the relationship between the
fat and cholesterol levels of most dairy products, but it 11. (D)
doesn’t address the difference between the two This one’s a method of argument question, so the
different products that form the basis of the paradox— operative question is: what’s the author doing? He’s
the non-dairy lightener with high fat, and the dairy presenting a belief, or proposal, and then telling us
product (milk with cholesterol). why, if the proposal were put in practice, it would have
illogical, foolish results. (D) is the closest paraphrase
9. (D) of this.
If the consumer is using a very small amount of (A) is a clear contradiction of what the author is trying
lightener, as opposed to a very large amount of milk, to do. True, he is stating a general principle, but he’s
then it follows that the amount of cholesterol in the presenting an argument against, not for, adopting it.
large quantity of milk will add up and have a greater (B) The author doesn’t offer specific evidence of
effect on the consumer’s blood cholesterol than the unfavorable consequences that have occurred—he
amount of saturated fat in the smaller quantity of offers his view about what would happen if the principle
lightener. So, if (D) is true, the manufacturers’ claims in the first sentence were adopted, but never provides
would be considerably strengthened. actual results of real applications of the principle.
(A) neither weakens nor strengthens the argument. (C) distorts the author’s main point. The thrust of the
We’re talking about changes in the typical consumers’ argument is not that the expected consequences won’t
levels, changes due to lightener or milk. The effect of result, but rather that unexpected ones will.
health practices on some people’s cholesterol levels is
(E) The author is upset about what would happen if
beyond the scope of the argument.
the principle were applied. Whether the principle can be
(B) The desserts that accompany coffee have nothing uniformly applied, or applied at all, is not in question.
to do with the effect of lightener, as opposed to milk,
on consumers’ cholesterol levels. 12. (D)
(C) is a useless distinction. Coffee lighteners that are The evidence: photovoltaic power plants, which
not based on coconut oil are irrelevant to the claims of produce electricity from sunlight, are now one-tenth as
manufacturers of lighteners that are based on coconut costly as they were twenty years ago, whereas
oil. traditional power plants have increased in cost. The
(E) Dismissing the possibility of psychosomatic conclusion: photovoltaic plants produce electricity less
effects, the beliefs of the consumer really don’t enter expensively than do traditional plants. But the fact that
into this discussion. one method is cheaper than it used to be while another
is more costly than it used to be is not enough to
10. (E) conclude that the first is therefore cheaper than the
This is really a thinly veiled formal logic stimulus. The second today. We need some link between the costs of
first sentence can be put into if-then form: If people the two methods that will allow the conclusion to
have serious financial problems, then they can’t be stand, and the assumption in (E) does the trick.
happy. And this, of course, is logically equivalent to its (A) simply restates some of the evidence.
contrapositive: if people are happy, then they do not (B) makes an irrelevant distinction. The amount of
have serious financial problems. electric power is not addressed in the stimulus; it’s the
(A) and (D) We can’t infer that serious financial cost of producing the power that we’re concerned with
problems (or in (A)’s case, serious problems—notice here.
the scope shift) are the only things that can make

11
PrepTest 3 Explained

(C) The author needn’t assume that none of the the past, and therefore, that the mental and emotional
advances can be applied to traditional plants in order strain of the long work hours may make them more
to conclude that photovoltaic plants produce electricity likely to make faulty decisions.
less expensively than do traditional plants. The main (A) is an au contraire choice. If the responsibilities of
issue is the relative cost of the two methods, and since the resident staff have not changed over the past
this doesn’t tie directly into that, it isn’t relied upon by decades, then Quincy seems right in arguing “if it ain’t
the stimulus. broke, don’t fix it.”
(E) So what? Unless we know what the cost of a (C) involves a scope shift: We’re concerned with medical
traditional plant is compared to that of a photovoltaic practice in general, not ER patients especially.
plant, we can’t say that the argument is properly
(D) makes a useless distinction. We’re not concerned
drawn.
with the differences in workload among the different
specialties.
13. (A)
(E) doesn’t give us enough information to effectively
Here’s another parallel reasoning stimulus that lends
counter Quincy’s argument. Although it argues for
itself to symbolic representation: If X (that insect is a
observation over thirty-six hours, and therefore could
bee), then Y (it can only sting once). Y (it only stung
support the need for residents to be on rounds for that
once), therefore X (it is a bee). This faulty structure is
long a period, we’re not told that the observation
matched by choice (A). It takes some rearranging, but
needs to be continuous.
the elements of (A) boil down to: If X (it is spring), then
Y (I cannot stop sneezing). Y (I sneezed), therefore X
15. (D)
(spring is here). Note how it exhibits the same flaw as
the original: In the stimulus, it’s very possible that the Experiments like this one are logically valid only if the
insect is not a bee, and is another insect that just two groups are exactly alike to begin with and if one of
happened to sting only once. In (A), it’s possible that them is exposed to one variable. In this case, the
it’s not spring, and that the person in question just variable is being shown violent TV programs right
happened to sneeze during some other season. before play. Since the author has concluded that the
experiment was valid, she is assuming that the two
(B) If X (the sky is clear), then Y (the atmospheric
groups had no differences other than the exposure to
pressure is high). Almost X (it’s clearing up), therefore
violent TV programs; in other words, that the television
almost Y (the pressure is bound to be high soon).
programs were the sole cause of the violence, and that
(C) If X (the painting is old and brittle), then Y (it will be nothing else could have been the cause. Choice (D)
moved with extreme care). Not Y (that painting is not correctly identifies this assumption.
moved with extreme care), therefore Not X (it is not old
(A) is a scope shift. The author is not talking about the
and brittle).
effect of all television programs on all of society. The
(D) If X (there was one more thunderstorm), then Y focus of this passage is the effects of violent television
(the roof would be ruined). Not Y (the roof is fine), programs on children.
therefore Not X (there must not have been any
(B) What if they’re not? Ultimate responsibility isn’t
thunderstorms).
the issue. In fact, the argument’s thrust is to show the
(E) Really doesn’t fit the structure we’ve set up. We responsibility of TV, not parents.
might be able to say: If X (one survives in the wild),
(C) is an au contraire choice. In order to make her
then Y (one has physical stamina like Mark’s). But then
argument, the author must believe that violence and
we get into Mark’s fear of spiders (Z?).
passive observation of violence (in this case, watching
violence on television), are directly related.
14. (B)
(E) is beyond the scope. The author never mentions
Quincy’s argument is that physician training does not
any violent treatment toward the children.
need to change because it has worked in the past. To
counter this, we need a choice that shows that current
16. (C)
medical practice is somehow different than in the past,
and therefore requires a change in training methods. Because waste gets disposed of in less populated
Choice (B) fits this qualification nicely. If (B) were true, areas, those who are responsible for dumping are not
it would mean that physicians in training would have to as fearless about its effects as they claim. This
deal with more crisis situations than did physicians in assumes that there is no plausible alternate

12
Section II: Logical Reasoning

explanation for the disposal pattern. (C) weakens the 18. (C)
argument by giving a reasonable alternative The author’s whole point is that Raghnall’s conclusion
explanation: Dumping nuclear waste in less populated is based on inadequate evidence. The author’s
areas poses fewer economic and bureaucratic evidence is the alternative explanation he provides for
problems than dumping in areas of denser population. the survey’s results; namely, that couples may blame
(A) and (B) are au contraire choices. The acknowledgment finances for their marriage problems when finances
that there could be an accident indicates that nuclear aren’t the real problem. He uses this alternative
waste does, in fact, pose some threat to people. explanation to make the point that Raghnall has
(D) is an irrelevant comparison. Pointing out chemical jumped to conclusions—that she has failed to consider
dangers won’t show that nuclear waste is safe; they’re other possible explanations for the survey’s results.
unrelated. Thus, the author believes that Raghnall’s conclusion is
inadequately justified.
(E) supports the author’s argument. It seems to be a
statement of what the policy makers really believe, but (A) distorts the argument. The author’s point isn’t so
just won’t admit. much that financial problems are not a big factor in the
breakup of marriages, but rather that Raghnall cannot
17. (C) reasonably conclude that they are without additional
evidence.
The United States has, overall, seen a decline in its
infant mortality rate in the past few years. But this (B) is outside the scope, a sure sign it’s not the main
does not mean that the babies born in the United point. Marriage counselors have never even been
States are healthier now than they were in the past. So mentioned.
the author is assuming the existence of an alternate (D) simply restates the evidence, and not even the
explanation for the decline in the infant mortality rate. author’s evidence, but Raghnall’s.
To support the argument, we need a choice that offers (E) is a subtle misreading. The author does allude to
this alternate explanation. Choice (C) tells us that the “a number of other articles,’’ but all we know is that
United States has developed technology that can save these ar ticles relied on the same sur vey that
babies that would have died otherwise. So while we Raghnall’s did; we don’t know that they necessarily
may have just as many sick or premature babies being drew the same conclusion that Raghnall did.
born, we have fewer babies dying as a result of
sickness or premature birth thanks to the advanced 19. (B)
technology. This would explain why the decrease in
We pretty much answered this in our analysis of 18:
mortality rates has no connection with the average
The author offers a different interpretation of the
health of the infants.
survey’s results—basically, that couples often express
(A) is useless background information; it doesn’t address their frustrations about other aspects of their marriage
the contrast between the overall infant mortality rate in financial terms, blaming money when money isn’t
and infant health. the problem. In other words, the author undermines
(B) doesn’t help us support the author’s claim; we Raghnall’s conclusion by offering an alternative
don’t need more information about infant death. What explanation for some of the data on which his
we do need is information that would explain why a conclusion was based.
decline in the infant mortality rate doesn’t signal an (A) The author never supplies us with a specific
increase in health. counterexample. He gives us an alternative
(D) We’ve already been told that the overall infant explanation, but he never gives us any specific
mortality rate has been declining. Like (A), this examples, such as: Couple X blamed money for their
provides background, but doesn’t do anything to problems, but their real problem was…
explain or support the claim that overall infant health (C) is off-base because all the author claims is that the
hasn’t improved. survey doesn’t establish that financial problems are
(E) is a “left-field” choice. We’re concerned with why the major problem in marriages. “Financial problems’’
health hasn’t improved along with the infant mortality or `”money’’ can hardly be called an emotion, and
rate; we’re not interested in the relationship between Raghnall hasn’t referred to anything else as a cause of
babies’ health and how much attention they receive. divorce.

13
PrepTest 3 Explained

(D) shifts the scope a bit: The author never criticizes (A) A healthier diet isn’t the issue here—we need
the survey; he criticizes Raghnall’s conclusion, which is information that fills in the connection between the
based on the survey. (E) goes too far. The author never blood cholesterol evidence and the conclusion of the
demonstrates or shows that couples cannot accurately Japanese researchers in the last sentence, and this
describe their marital problems. He merely claims that isn’t it.
they often fail to do so. (C) The author tells us that high blood cholesterol
lowers the risk of weakened artery walls. He never
20. (C) says that it eliminates this risk, nor does his argument
Since we’re asked to pick the one choice out of five depend on this information.
that does NOT weaken the argument, we know that the (D) is an irrelevant comparison. In fact, the author
argument will be pretty vulnerable to weakeners. There could assume that cerebral hemorrhages are less
are a number of assumptions at work in this argument, dangerous than strokes caused by blood clots, and it
many of which lead to the weakeners in the wrong wouldn’t damage his claim that Western diets are less
choices, but choice (C) is correct because it’s totally likely to lead to cerebral hemorrhages.
irrelevant to the issue of honey production.
(E) involves a scope shift: Low blood pressure is an
(A) weakens the argument by undermining the major irrelevant issue, because we don’t know how this
assumption that what’s true of Brazil will be true of the relates to blood cholesterol.
U.S. If, as (A) claims, the native bees in Brazil are
different from the ones here, then the comparison the 22. (A)
author cites is irrelevant—maybe domestic U.S. bees
This is simply a matter of a very common chain of
produce more honey than both Brazilian and
argument: IF X, then Y. If Y, then Z. Therefore, if X,
Africanized bees do.
then Z. Specifically: If the country is to remain
(B) and (D) both undermine another one of the internationally competitive, then there is an undeniable
argument’s basic assumptions: that commercial honey need for citizens to better understand international
production won’t decline for some other reason. If, as affairs. If there’s a need for this better understanding
(B) says, it’s more costly and difficult to use of international affairs, then all of our new teachers
Africanized bees, or if, as (D) says, a lot of the people must be prepared to teach their subject matter with an
now responsible for honey production would rather international orientation. This allows us to infer that
cease and desist than use Africanized bees, then it’s the first statement leads to the third: If the country is
quite possible that commercial honey production will to remain internationally competitive, then all of our
decline if these bees are introduced. In any case, the new teachers must be prepared to teach their subjects
author can no longer conclude for sure that it won’t with an international orientation.
decline.
(B) and (C), like many choices before them on this
(E) If Africanized bees are better suited to Brazil, then test, confuse necessary and sufficient conditions. The
the author can no longer assume that in America, stimulus establishes the ability of new teachers to
they’ll produce more honey than American honeybees. teach with an international focus as necessary for the
Who knows, maybe they’ll all die off once they get country to remain competitive. We can’t infer, as (B)
here. suggests, that it’s sufficient for this. And as for (C),
better understanding of international affairs by our
21. (B) citizens is also necessary, but not sufficient, for the
According to a recent report, low blood cholesterol country to remain competitive.
weakens artery walls, increasing the likelihood that the (D) completely denies the author’s first if-then
arteries will rupture, and thereby bring about a cerebral statement, which is an important premise in the
hemorrhage. The author concludes that this new report argument.
supports the long-held belief of Japanese researchers
(E) All we’re told about public reports is that they
that Western diets are better at protecting against
stressed the need for citizens to better understand
cerebral hemorrhage than are non-Western diets.
international affairs. We have no evidence that they
For this conclusion to be valid, the author must be said anything about training teachers to teach with an
assuming that Western diets lead to a higher blood international focus—that requirement was inserted by
cholesterol level than non-Western diets. the author.

14
Section II: Logical Reasoning

23. (C) (E) A similar story: this choice interprets independence


We’re asked to weaken the proponents’ argument, and as sufficient for progress, when in fact, all we’ve been
conveniently enough, we’re given their assumption. told is that it’s necessary. Certainly, there are other
Your best bet is to look for a choice that undermines factors that could come into play.
that assumption, which we get in (C): If genetic
characteristics can occur in sets, that breaks down the 25. (E)
proponents’ assumption that these characteristics The key here is the author’s vague use of the term
occur independently. “adequate.” If our main enemy is gone, who’s to say
(A) is an au contraire choice. If the genetic material that a reduced defense budget wouldn’t be adequate?
that all people have in common with each other and If the primary motivation for spending so much money
with animals is excluded from the procedure, then that on defense in the first place is no longer relevant, isn’t
makes the analysis much more specific, and, inferably, it reasonable to expect that we could spend less
more valid. money and still defend ourselves as “adequately” as
before?
(B) If anything, this choice strengthens the proponents
claim by dismissing a possible argument against the (A) distorts the passage—since the whole point is that
procedure; the argument that it’s unreliable because the public may no longer be persuaded, the author
different people could obtain different readings of the certainly doesn’t argue that this “manipulation’’ can
same pattern. continue indefinitely.
(D) contains a scope shift: The point here is whether (B) just plain denies what the author says, which does
the procedure is valid when done accurately. not amount to finding a weakness in his reasoning. We
want a criticism of his reasoning, not merely an
(E) is irrelevant. The issue is whether or not DNA
unsupported denial of his facts.
fingerprinting can be accurately used to match two
different samples of genetic material. Any other use is (C) is a fancy way of accusing the author of using
a separate issue. circular reasoning. However, the evidence (the
dissolution of the Eastern bloc) is quite different from
24. (A) the conclusion, which is that there will be insufficient
support for an adequate defense budget.
The argument compares schools to cultures; schools
would have to progress the same way cultures (D) The author does give a reason for his opinion: the
progress. Cultures cannot progress if outsiders impose dissolution of the Eastern bloc. It may not be a very
their views. So the same must be true for an individual good reason, but it’s a reason all the same.
school: If a school is to progress, it must be free of
outside imposition. Choice (A) carries out this analogy
between schools and cultures.
(B) The author doesn’t mention degrees of independence,
or degrees of initiative; he merely claims that the only
way a culture can progress is if it is independent of
outside imposition.
(C) We’re told that progress for an individual school, or
culture, requires independence from outside
imposition. The idea of school system officials deciding
what changes to make is directly counter to this.
(D) goes too far. The author never says outsiders must
be totally shut out if progress is to be achieved; in fact,
he admits they can provide valuable advice.

15
PrepTest 3 Explained

SECTION III: • Although the author’s specific main idea isn’t


entirely clear until you’ve read through the whole
READING COMPREHENSION passage, topic, scope, and purpose are all
revealed by line 15. That makes this passage an
Passage 1: Asteroids ideal place to start work on this RC section, even
if you suffer from “science anxiety.” Topic, scope,
Questions 1–7 and purpose, after all, are the three things that
you need to grasp as quickly as possible in order
Topic and Scope: Asteroids; specifically whether or not to get the passage under control.
it’s possible for asteroids to have satellites.
The Questions:
Purpose and Main Idea: The author examines the state
of the evidence for asteroid satellites. While 1. (E)
observations of “secondary occultations” have led This passage is about the existence of asteroid-satellite
some astronomers to accept the existence of such systems: astronomers used to doubt their existence,
satellites, others remain unconvinced and are looking but now recognize that they are theoretically possible,
for specific further evidence (“well-behaved” secondary and are looking for definitive proof. (E) correctly notes
events). that theoreticians were on to the existence of asteroid-
Paragraph Structure: Paragraph 1 introduces the satellite systems before astronomers were. Moreover,
topic, saying that while most astronomers used to (E)’s idea that astronomers agree on what would be
think it was impossible for asteroids to have satellites, conclusive proof echoes the final paragraph.
theoreticians knew all along that such a thing was (A) focuses on a detail—the Heruclina event. But the
possible. Paragraph 2 supports the theoreticians; passage is about asteroid-satellite systems in general,
observations have led many astronomers to believe not just about the Herculina observations.
that asteroids can have satellites, because when (B) Au contraire. The first sentence of the passage
asteroids pass in front of stars, “something besides tells us that astronomers “long believed” that stable
the known asteroid sometimes blocks out the star as asteroid-satellite systems weren’t possible; furthermore,
well.” skepticism about the existence of such systems has
Paragraph 3 describes the “most convincing…report” decreased.
of an asteroid that might have a satellite. When the (C) Au contraire aussi. The Herculina event supported
asteroid Herculina passed in front of a star, the the theoreticians’ views about asteroid-satellite
occultation was preceded by another occultation, which systems. Besides, like (A), this choice is focused on a
led astronomers to believe that a satellite orbiting the detail.
asteroid had also passed in front of the star. Paragraph (D) is a “half-right, half-wrong” choice. Skeptical
4 notes that, after the Herculina event, reports of astronomers aren’t waiting for new theoretical models,
secondary occultations became “respectable,” but but for physical evidence in the form of photoelectric
warns that such reports have grown so numerous that records.
they can’t possibly all be accurate. Paragraph 5
concludes by saying that even astronomers who remain 2. (D)
skeptical would be convinced by a “photoelectric
record” of a “well-behaved” secondary occultation of a Heruclina is discussed in Paragraph 3. It says there
star, one definitely caused by a body accompanying an that astronomers watching the Herculina event were
asteroid. surprised by an unexpected drop in brightness that
occurred before the drop they were expecting. That’s
The Big Picture: (D): evidence of Herculina having a satellite was
provided by “the occultation that occurred shortly
• A firm grasp of the passage doesn’t mean
before the predicted occultation by Herculina.”
absorbing all of its details (you can look them
up if you need to). Rather, it means figuring out (A) First, the secondary body’s presence was “strongly
what the author’s doing in the text—in this indicated,” not “directly observed.” Second, what’s
instance, providing evidence that suggests that important is the fact that a second body also eclipsed
theoreticians have been correct all along about the star.
asteroid-satellite systems. (B) is outside the scope. The passage says nothing about
a planet near Herculina.

16
Section III: Reading Comprehension

(C) is also outside the scope. The author never discusses what constitutes a “well-behaved” event only arises
the amount of time needed to complete an orbit. later (in Paragraph 5).
(E) The occultation of Herculina itself is irrelevant; the (E) Au contraire aussi. Prior to the Herculina event, it
evidence for the existence of a satellite was provided wasn’t respectable to report the observation of a
by the secondary occultation. secondary event.

3. (A) 5. (D)
The attitude of astronomers since the Herculina event Paragraph 4 says that repor ts of secondar y
is discussed in paragraphs 4 and 5. Paragraph 4 says occultations grew so common after the Herculina event
the Herculina event made secondar y sightings that they’re now too numerous for all to be correct.
“respectable”—i.e many astronomers came to accept Why? Because even if every asteroid has the highest
the possible existence of asteroid-satellite systems. plausible number of satellites, “only one in every
Paragraph 5 indicates that even astronomers who are hundred primary occultations would be accompanied
still doubters would be convinced by the right kind of by a secondary event.” So (D) must be correct: Since
evidence. As (A) puts it, astronomers who were the Herculina event, reports of secondary events have
skeptical of the existence of asteroid-satellites have been occurring at a rate greater than this maximum
become more open-minded, although many are still plausible rate of one in every one hundred cases.
awaiting proof. (A) and (C) simply can’t be concluded based on the
(B) describes the attitude of many astronomers prior to passage’s information. We don’t know how many
the Herculina event. repor ts of primar y occultations have included
(C) There’s no “chaotic mix of theory” relating to secondar y occultations, or how many reports of
asteroid satellites; paragraph 1 indicates that a single, secondary occultations there are or were, so we can’t
simple theory supports their existence. Further, the calculate the increase of either.
passage doesn’t say that any data is “spurious.” (B) distorts the last parenthetical clause of paragraph
(D) is too negative. Many astronomers already believe 4, which describes what would be the case if asteroid-
that asteroid satellites exist. The skeptics are merely satellite systems resembled planet-satellite systems,
looking for one particular kind of evidence; they aren’t not what is in fact the case on actual reports of
rejecting “all data not recorded automatically by state- secondary occultations.
of-the-art instruments.” (E) is beyond the scope. The passage never mentions
(E) There’s nothing in the passage about admiration any repor t containing more than one secondar y
for the scientific process, nor has there been occultation.
“incontrovertible proof” of anything.
6. (C)
4. (C) (C) is right on the money: the author’s primary purpose
(C) gets to the heart of the matter. The first sentence is to trace the development of ideas among
of paragraph 4 says that after the Herculina event astronomers concerning the existence of asteroid-
secondary occultations became “‘respectable’—and satellite systems.
more commonly reported.” This implies that before the (A) Much of the passage describes how reporting
Herculina event secondar y occultations weren’t “secondary occultations” has become respectable.
considered respectable, and so weren’t commonly (B) deals only with information in paragraph 5. Moreover,
reported. (B)’s distinction between “spurious” and “theoretically
(A) Au contraire. Paragraph 1 indicates that a good believable” observations isn’t made in the passage.
theoretical model of asteroid-satellite systems did (D) The author isn’t trying to bring a “theoretician’s
exist prior to the Herculina event. perspective” to the discussion; instead, he’s primarily
(B) The author never implies that satellite collisions interested in how experimental results enlighten the
were mistaken for occultations. There’s no speculation disucssion.
on what, other than an actual satellite, might have (E) is easy to eliminate, because it never even
occasioned the rare repor ted obser vations of mentions asteroid satellites. Moreover, there’s no
secondary events before the Heruclina event. attempt to limit speculation about occultation.
(D) Prior to the Herculina event, it wasn’t even
respectable to report secondary events. The issue of

17
PrepTest 3 Explained

7. (C)
Based on paragraph 5, you can expect the answer to
have something to do with the photoelectric record. (C)
would provide the hard physical evidence we need; a
photoelectric record of a “well-behaved” secondary
event is exactly what skeptical astronomers say “would
change their minds.”
(A) The existence of such early reports is only hinted
at, and the clear implication is that nobody took them
very seriously.
(B) The author never implies that there’s anything
wrong with the original theoretical model.
(D) is just a more refined abstraction — it doesn’t
constitute the kind of physical proof needed to resolve
the question.
(E) distorts the passage’s final sentence, which
suggests that airplanes passing in front of the
instruments might be responsible for some
observations of secondary events.

18
Section III: Reading Comprehension

Passage 2: 17th-Century Scientific The Questions:


Discovery 8. (C)
(C) covers the gist of the passage. The author explains
Questions 8–15
how much of 17th-century experimental work was
Topic and Scope: Scientific experimentation in 17th- performed by technicians, and how that work was
centur y England; specifically, who per formed denigrated and distrusted for the reasons discussed in
experiments and why. paragraph 3.
(A) ignores the major concern of the passage: why
Purpose and Main Idea: The author seeks to show how technicians’ contributions were overlooked. Moreover,
the social prejudices and scientific views of 17th- the passage never implies that scientific experiments
century English scientists led them to leave much of would have been absolutely impossible without the aid
their lab work to technicians, and prevented them from of technicians.
giving technicians the proper credit for their work.
(B) distorts the passage. Lab workers, as salaried
Paragraph Structure: Paragraph 1 states that a employees, belonged to a lower social caste.
distinction must be made between the way scientific (D) is outside the scope. The passage never discusses
experimentation was described by 17th-century English the relationship between 20th-century scientists and
scientists and the way it was actually performed. In their technicians.
theory, 17th-century scientists believed that experiments (E) focuses on a detail.
should be performed by the scientists themselves,
without relying on others for assistance. Yet, as 9. (E)
paragraph 2 makes clear, scientists often did not act in
Seventeenth-centur y rhetoric about scientific
accordance with their beliefs. As the example of Robert
experimentation emphasized the idea that scientists
Boyle shows, many of them were aided in their
should do their own experiments. Hence, that rhetoric
experiments by paid technicians whose contributions
would have more accurately described the work
went unacknowledged.
conducted in Boyle’s lab if Boyle himself had actually
Paragraph 3 gives three reasons why the role of done his own experiments.
technicians was unacknowledged: (1) the belief that
(A) would contradict seventeenth-century rhetoric by
scientific breakthroughs occur as a result of flashes of
having Boyle admit that he relied on others.
insight on the part of brilliant individuals rather than
through group efforts; (2) the fact that 17th-century (B) According to the passage, contempt for manual
English scientists were members of the upper class labor was characteristic of seventeenth-centur y
who held the manual labor done by their technicians in scientists.
disdain; and (3) the tendency to disregard as unreliable (C) Membership in the Royal Society alone wouldn’t
the input of the wage-slave technicians. determine whether or not Boyle performed experiments
according to the rhetoric of the day.
The Big Picture:
(D) focuses on the wrong issue. The rhetoric in question
• Since topic, scope, and purpose all appear in the was about scientists per forming all their own
first sentence, this passage is another candidate experiments, not about acknowledging technicians;
to be worked on early in the section. acknowledging technicians is an issue raised by the
• Keywords can help you negotiate a passage author.
easily. The long third paragraph is neatly divided
by keywords. The question at the beginning 10. (C)
introduces the paragraph’s focus, and the three The “franchise” and it’s relation to ser vants is
answers given are clearly marked by the words discussed in the middle of paragraph 3, where it’s said
“one reason,” “moreover,” and “finally.” that ser vants were excluded from the franchise
because, as wage earners, they were thought to be
controlled by their employers. In other words, as (C)
says, their political independence was thought to be
compromised.

19
PrepTest 3 Explained

(A) Servants weren’t excluded because their interests (E) states a position that certainly wasn’t endorsed in
were already represented, but because it was believed the seventeenth century (and may not be widely
that they would blindly support the political positions of endorsed in the twentieth century, for all we know).
the employers who paid their wages.
(B), (D), and (E) are outside the scope. Nowhere does 13. (D)
the passage state or imply that ser vants were Paragraph 3 begins by asking why the role of
inadequately educated (B), a polarizing force (D), or technicians wasn’t acknowledged by 17th-centur y
insufficient contributors to society (E). scientists, an issue introduced in Paragraph 2. It then
answers this question by discussing three factors that
11. (D) contributed to the failure to acknowledge the role of
In the middle of Paragraph 1, we’re told that the Royal technicians.
Society of London endorsed the notion that doing (A) The question isn’t posed in the previous
menial work in the cause of science was a good thing. paragraph, nor is only one of several alternative
As (D) puts it, the Society advocated abandoning the answers adopted.
traditional upper class ethic against per forming (B) None of the factors discussed in paragraph 3 is
manual labor—at least as far as science was rejected.
concerned.
(C) The explanations discussed aren’t incompatible—
(A) and (E) are au contraire choices. Pararagraph 1 the factors in paragraph 3 are shown as working
explicitly says that, as far as the Royal Society of together.
London was concerned, the willigness of scientists to
(E) Paragraph 3’s explanation doesn’t rest on recent
do their own manual labor was part of an attempt to
research, but is based on the author’s interpretation of
discover God’s truth in nature (A), as well as a
seventeenth-century English society.
demonstration of piety (E).
(B) and (C) touch on issues which the Royal Society 14. (A)
didn’t address. The Society never asked scientists to
In introducing the political significance of the wage
abandon the individualistic view of scientific
relationship, the author makes the point that workers
breakthroughs (B) or the view that wage-dependent
dependent upon the wages of their employers simply
servants shouldn’t vote (C). In fact, based on the
weren’t considered reliable, whether in political
information in the passage, it’s quite likely that the
judgments or in scientific research. As (A) says, the
upper class Royal Society would have endorsed these
author puts the scientists’ failure to acknowledge the
views.
contributions of technicians in the context of general
worker-employer relations.
12. (A)
(B) is outside the scope. The author offers no general
The second sentence of paragraph 3 indicates that
thesis about the relationship between scientific
both the seventeenth and twentieth centuries share
discovery and economic conditions.
the view that scientific discoveries result from the
sudden insights of a small number of brilliant (C) goes against the gist of the passage, which is that
individuals, rather than from the cooperative efforts of seventeenth-century scientists relied on technicians to
many people. do much more than simply the most menial tasks.
(B) is outside the scope. There’s nothing in the passage (D) is also outside the scope. The author doesn’t
about either seventeenth or twentieth century views of discuss political or economic changes in seventeenth-
the connection between political values and scientific century England.
method. (E) The author doesn’t tie the wage relationship to the
(C) and (D) are out because, while the beliefs that nature of scientific discovery, but to the attitude of
research undertaken for pay couldnt be objective (C) scientists towards technicians.
and that scientific discovery could reveal divine truth
(D) were indeed both held in the seventeenth century,
the passage says nothing about twentieth-century
beliefs concerning these things.

20
Section III: Reading Comprehension

15. (D)
As we’ve already seen, the rhetoric of seventeenth-
century English science concerned the idea of doing
hands-on research. What was the rhetoric? That
scientists should conduct, observe, and analyze their
own experiments.
(A) The myth about how discoveries were thought to
occur is a general trend discussed by the author, not a
subject of seventeenth-century rhetoric.
(B) Au contraire. Seventeenth-centur y scientific
rhetoric emphasized the importance of doing manual
labor in the cause of science.
(C) Seventeenth-century scientists failed to acknowledge
the contributions of their technicians. The scientific
rhetoric of the time never addressed that issue.
(E) Though scientists like Boyle did believe in the
search for divine truth in nature, the “rhetoric”
discussed in the passage concerned only the
importance of manual labor in research.

21
PrepTest 3 Explained

Passage 3: Monopoly Power The Questions:

16. (E)
Questions 16–20
Paragraph 5 says that antitrust laws focus on abuse of
Topic and Scope: Monopoly power; specifically, the monopoly power rather than possession of it in order to
difference between the possession of monopoly power, protect consumers’ welfare. So we can infer (E): that
which isn’t illegal, and the abuse of monopoly power, the abuse of monopoly power is prohibited because it
which is. impairs consumer welfare, whereas possession
doesn’t necessarily hurt consumers.
Purpose and Main Idea: The author’s purpose is to
(A) The author doesn’t make a legal distinction
describe what sort of exercises of monopoly power are
between market share and market control.
considered violations of federal antitrust laws. Since
this is a descriptive passage, there really isn’t a (B) The author never suggests that monopoly power is
specific main idea. easier to demonstrate than abuse.
(C) In the first paragraph, abuse of monopoly power is
Paragraph Structure: Paragraph 1 explains that the defined as the exclusion of competition “in the
possession of monopoly power is not in itself illegal; to monopolized market or related markets,” so it needn’t
violate antitrust laws, a company must abuse involve more than one market.
monopoly power by using it to exclude competition.
(D) Paragraph 3 says that charging supracompetitive
Paragraph 2 explains how monopoly power comes prices doesn’t by itself constitute an abuse of
about: Companies with a large market share can raise monopoly power.
prices above competitive levels without losing
customers. Paragraph 3 explains why the mere 17. (E)
possession of monopoly power isn’t illegal: Tighter
At the beginning of paragraph 4, leverage is described
laws might pose disincentives to the growth of
as “the use of power in one market to reduce
monopolies and impair consumers’ welfare. Paragraph
competition in another,” a strategy which is clearly
4 describes the types of exclusionary practices which
characterized as abuse in paragraphs 1 and 4.
constitute abuse.
(A) Au contraire. We’re told that the manipulation of
Paragraph 5 reiterates a point made earlier—that, in
related markets constitutes abuse, even though these
the interests of consumer welfare, antitrust laws focus
secondary markets aren’t monopolized.
on the abuse rather than the possession of monopoly
power. (B) “Tying arrangements” are presented as an example
of leverage strategy, and all leverage strategies are
The Big Picture: considered abuses by the author.
• Be on the lookout for passages that contrast two (C) Au contraire aussi. The use of monopoly power in
or more entities: the possession vs. the abuse of itself doesn’t constitute abuse.
monopoly power, for instance. Such passages (D) A company using leverage would still violate antitrust
always have questions that hinge on a clear laws, even if it was charging competitive prices.
understanding of the difference between the
entities being compared. 18. (D)
• You don’t have to assimilate all of the details to The passage as a whole revolves around the distinction
do well on this passage. The important thing is to between possession of monopoly power and its abuse.
understand the basics of monopoly power— In the third paragraph, the author brings up a number
what’s illegal (abuse), what’s not (possession) of cases where companies that possess monopoly
and why (consumer welfare). power use it legally. The author is clarifying how far
companies can legally exercise monopoly power. As (D)
puts it, the author is distinguishing what is covered by
the antitrust laws from what isn’t.
(A) is outside the scope. There’s no mention of
supracompetitive profits in the passage, if indeed such
profits exist.

22
Section III: Reading Comprehension

(B) distorts the passage. We’re told how far companies (D) dredges up the idea of “supracompetitive” profits,
can exercise monopoly power without breaking the law, which aren’t mentioned anywhere in the passage.
a quite different thing from describing positive uses of (E) also goes against the gist of the paragraph.
monopoly power. Restraints on monopoly haven’t been left to the
(C) focuses on a detail, not the main purpose of the market, but rather have been enforced by antitrust laws.
paragraph.
(E) doesn’t really emerge until paragraph 5; it’s by no
means the central idea of paragraph 3.

19. (B)
Essentially the lawmakers’ attitude toward monopoly is
that some methods of reducing competition are
legitimate, and some aren’t.The point of the passage,
after all, is to distinguish between legal and illegal
forms of monopoly. Paragraphs 2 and 3 center around
the extent to which companies can exercise monopoly
power without violating antitrust laws. Paragraphs 4
and 5, on the other hand, focus on uses of monopoly
power that are prohibited by antitrust laws.
(A) Au contraire. At the end of paragraph 3, we’re told
that monopolist companies can be allowed to grow at
the expense of competition in the interests of
consumers’ welfare.
(C) The author says that consumer welfare is the
principle aim of the antitrust laws.
(D) Au contraire aussi. According to paragraph 2, when
close substitutes for a product are available,
competition benefits from a company that charges
supracompetitive prices.
(E) Since the existence of monopolies is considered
better for consumer welfare under cer tain
circumstances, lawmakers presumably wouldn’t agree
that competition is necessary to supply high-quality
products at low prices.

20. (A)
The author’s point in the final paragraph is that the
legal distinction between possession of monopoly
power and its abuse is based on a desire to promote
consumer welfare. We want a choice that’s relevant to
this idea. (A) fits the bill by picking up the consumer-
welfare-based distinction between abuse and
possession of monopoly power, and explaining that
monopoly power can sometimes be in the consumers’
best interests.
(B) goes against the gist of the last paragraph. The
author believes that antitrust laws have been effective
in securing the consumers’ best interests.
(C) is outside the scope. It focuses on two particular
industries that haven’t been mentioned anywhere.

23
PrepTest 3 Explained

Passage 4: Navaho Blankets The Questions:

21. (E)
Questions 21–28
This choice captures the passage’s focus on Amsden’s
Topic and Scope: Navajo weaving; specifically, the views of Navajo weaving styles and the author’s
different styles of Navaho rug weaving and how they critique of those views.
developed. (A) is outside the scope. The author never suggests
that the Navajo rejected all Anglo cultural influence.
Purpose and Main Idea: The author describes a
theory—Amsden’s theory about Navajo weaving styles (B) distor ts the author’s criticism. The author
and how they evolved—and then calls that theory into questions Amsden’s account of how the styles
question. developed, but doesn’t reject Amsden’s categorization
of the styles.
Paragraph Structure: Paragraph 1 introduces (C) focuses on a detail. It plays on an idea that the
Amsden’s view of Navajo weaving styles: three of them author puts forth in paragraph 5.
are banded with stripes, zigzags, or diamonds, while
(D) is outside the scope as well. It doesn’t even
the fourth style is quite different, a border surrounding
mention Navajo weaving, Amsden, or the author.
central figures. Paragraph 2 explains that Amsden
believes that there’s some Anglo influence in the
22. (A)
diamond style, but the most Anglo influence appears in
the bordered style. Paragraph 3 gives the meat of The author mentions the strips of color breaking
Amsden’s argument: he believes that the bordered rug through the enclosed border as evidence of Navajo
represents a radical break with previous styles, and distaste for the Anglo preference that graphic designs
that the very fact of the border changed the way Navajo have a top, bottom, and border. (A) paraphrases this
weavers designed rugs. sentiment, albeit in abstract language.
Paragraph 4 begins the author’s criticsm of Amden’s (B) Amsden depicts the strips of color as signs of
theory: “Amsden’s view raises several questions.” general Navaho abhorrence for borders, not
First question: what is involved in altering artistic necessarily an “echo” of the diamond style.
styles? The author concludes that in the case of (C) Au contraire. For Amsden, the strips of color
weaving, there’s no radical change in motor habits or bursting through the border reflect resistance to Anglo
thought processes. Paragraph 5 raises the second culture.
question: what’s the relationship between banded and (D) According to Amsden, the Navajo resisted the
bordered styles? The author contends that the break in bordered style, not the banded style.
style isn’t a break in psychology, but a result of the
(E) The desire for designs with a top, bottom, and
ar tist’s quest for invention. Finally, Paragraph 6
border is presented as an Anglo desire.
questions the idea that there really is a stylistic gap
between banded and bordered styles.
23. (C)
The Big Picture: (C) captures the author’s view as it is expressed in
• When more than one view is presented, you need paragraphs 5 and 6. The author thinks that the
to be clear about the distinctions between or bordered style gradually evolved from the banded style
among the different points of view. The questions not necessarily as a result of Anglo influence, as
will certainly test to see that you’ve grasped the Amsden believed, but as a result of Navajo
differences. experimentation with design and the artistic quest for
invention.
• Notice how neatly this passage is arranged. The
first three paragraphs describe Amsden’s views, (A) In mentioning the Chief White Antelope blanket at
while the last three—beginning at line 28— the end of the passage, the author suggests that the
supply the author’s critique of Amsden. diamond style pre-dated the arrival of the Anglos.
(B) is outside the scope. There’s no evidence of what
the author thinks is “generally” the case when two
cultures occupy the same region.

24
Section III: Reading Comprehension

(D) is also outside the scope. Nothing in this passage (A) is clearly incorrect because Amsden sees little or
suggests that non-Anglo cultures influenced Navajo no correspondence between Anglo and Navajo art.
weaving. Rather, he views the two styles as radically different.
(E) Saying that vertical arrangements of diamond (B) is outside the scope. The author never discusses
parts “anticipated the border” isn’t the same as saying Amsden’s feeling about Anglo culture.
that “rows” of horizontal and vertical diamonds were (D) The author doesn’t criticize Amsden for basing his
“transformed into solid lines” to create the border. theories on a limited number of weaving specimens.
(E) is also outside the scope. There is no suggestion
24. (D)
that Amsden has confused the features of the zigzag
What happened in 1890? The bordered style appeared. and diamond styles.
So, you’re looking for the choice that’s not characteristic
of pre-1890 weavings—i.e., that’s uniquely 27. (B)
characteristic of the bordered style. The bordered style
In the final paragraph, the author makes the point that
used isolated figures (paragraph 1), while pre-1890
some stylistic changes that led the way to the border
weavings used continuous patterns (paragraph 3).
style can’t be attributed to Anglo influence, and uses
(A) Paragraph 3 says that the old patterns alternated the Chief White Buffalo blanket as an example to
decorations like stripes, zigzags, or diamonds in a illustrate this point.
regular order—that’s a “repetition of forms.”
(A) Au contraire. The Chief White Antelope blanket
(B) Early Navajo rugs were continuous, with overall argues against the influence of Anglo culture on the
patterns rather than isolated figures. bordered style.
(C) and (E) According to paragraph 1, “horizontal bands” (C) Au contraire, too. The author says that the
and “color” were used in early styles. ver tically arranged diamonds in the Chief White
Antelope blanket anticipate the border. Moreover, this
25. (D) blanket has a “flowing design,” not a central design.
The author accepts Amsden’s classification that (D) The Chief White Antelope blanket questions the
Navajo weavings used horizontal bands of abstract idea of Anglo influence.
designs early on, and later moved on to isolated figures
(E) This blanket seems to illustrate innovation within
(when the bordered style was adopted); but differs
the diamond style.
from Amsden in arguing that the change was gradual,
not a radical break.
28. (B)
(A) In Paragraph 4, the author suggests that motor
Paragraphs 1 through 3 describe Amsden’s view about
habits and thought processes have little application to
how the bordered Navajo weaving style developed.
Navajo weaving.
Paragraphs 4 through 6 question that view. Thus, this
(B) Neither the author nor Amsden attributes the zigzag choice accurately sums up the passage’s primary
style to Anglo influences. concern.
(C) Au contraire. The figures came later. (A) Although the passage does compare different
(E) Also au contraire. Only the border style is said weaving styles—the banded and the bordered—the
routinely to contain isolated figures. central concern is not to compare the styles, but to
question a view regarding the development of these
26. (C) styles.
The author’s main point of issue with Amsden concerns (C) is outside the scope. The passage never proposes
the claim about Anglo influence. The author’s objections new methods of investigation.
occur in paragraphs 4, 5, and 6. The author’s basic (D) This choice focuses on a detail. While the author
point here is that Amsden has overlooked some does discuss the influence of Anglo style on Navajo
things—the nature of weaving, the artists’ quest for weaving, that’s done only in order to question
invention, the existence of intermediate forms—that Amsden’s explanation of how the bordered style
suggest that the bordered style may have arisen without evolved.
Anglo evidence. As (C) says, the author thinks that
(E) The author’s focus is on the evolution of a style.
Amsden fails to consider certain aspects of Navajo
The interaction between two cultures is an idea of
weaving in making his claim.
Amsden’s.

25
PrepTest 3 Explained

SECTION IV: (A) asserts that computers are as good as teachers in


drilling students on rules and facts, and (D) goes one
LOGICAL REASONING step further and says that they’re better. The author
freely admits that computers would be capable of
1. (D) replacing teachers if that’s all there is to it, but argues
The only way for a violin to play recognizably the same that computers fall short because learning requires a
music as the piano is for the violinist to keep the grasp of underlying concepts as well.(B) Au-contraire;
nature and possibilities of his instrument in mind, and this actually supports the argument by stating that the
to try to suit the original composition to the new teacher’s essential task is to make students
instrument. Since the two different instruments are understand the general concepts behind specific facts
intended to be analogous to the two different and rules—exactly what the author claims computers
languages, the point of the argument must be that are not able to do.
translation from one language to another can only be (E), if anything, supports the argument, because the
achieved if the translator does not simply try to copy author implies that drills and coaching are the
the original exactly, but is also guided by the innate computer’s strong point.
possibilities and limitations of the new language.
(A) No; by the very nature of the analogy, the author is 3. (C)
implying that poetry tranlation is possible, so long as Here’s the reasoning: If city council spending (X)
the translation is guided by the nature of the language remains the same, then the sales tax (Y) can be
it’s translated into. expected to be at a certain level. Therefore, if the sales
(B) attempts to confuse the issue by combining the tax (Y) is higher than the predicted level, then it will be
two elements of the analogy, music and poetry, byt the because city council spending (X) increased. This
argument is not even remotely concerned whether element of causation throws a kink into the logic. The
some languages are more or less musical, and hence contrapositive of the first sentence is “If the sales tax
more or less poetic, than others. It also strays from the is not 2%, then city council didn’t maintain spending at
scope, translation. the same level.” The higher tax mentioned in the
(C) also tries to combine poetry with musical qualities second sentence could be the result of other factors; it
like rhythm and sound patterns, therefore confusing need not be caused by an increase in expenditures. (C)
the purpose of the analogy, which isn’t included to most closely parallels this, complete with the logical
convince us that poetry should be like music, but flaw: If workers’ wages (X) aren’t increased, then the
rather to suggest that similar factors that influence prices charged for goods (Y) will remain at the level it
musical translation may also influence poetr y was at last year. Therefore, if the prices (Y) rise beyond
translation. the predicted level, then it will be because the workers’
wages (X) also increased.
(E) is plainly outside the scope of the argument, which
never debates the relative difficulty of translating (A) begins with a premise relating the cost of house-
philosophical insights or subjective impressions. building to the price of houses, but goes on to try to
deduce something about builders attempting to sell a
2. (C) greater number of houses, which is a completely new
idea.
According to the author, if a student’s understanding of
a subject consisted only of knowing facts and rules, (B) also complicates things by introducing the idea of
then computers might well eventually replace human reduced profits due to shoplifting, a third term in the
teachers as drill masters and coaches. However, a argument.
student’s understanding also consists of having a (D) Much like in (B), (D)’s conclusion bears no
grasp of the general concepts underlying them. The resemblance to the stimulus conclusion, since it
author concludes that the computer will not eventually introduces the totally new idea of improved services, a
replace the teacher, obviously assuming that third term not found in the original.
computers can’t teach these general concepts along (E) is concerned with qualitative terms, whereas the
with the facts and rules. (C) undermines the argument stimulus was concerned with quantitative terms. (E)
by attacking this assumption. also drags a new consideration, newspaper circulation,
into its conclusion.

26
Section IV: Logical Reasoning

4. (D) observation about the double effect of clocks and


Good deeds are beneficial to the immune system of the applying it to new machines in general.
person who does them. White blood cells are needed (B) is never suggested, and besides, we’re looking for
to fight infection, and magnanimous behavior causes a general concept that can be illustrated by the clock
the brain to produce chemicals that stimulate and aid example, not a recommendation of what to do.
the activity of these white blood cells. We are therefore (C) The clock example doesn’t match this at all, as it
safe in inferring that magnanimous behavior is illustrates that new technologies can improve our lives,
beneficial to one’s own interests. even those that also have a restricting effect.
(A) The stimulus says nothing about what constitutes (D) The author does not offer a final judgment about
a good deed, or what sort of motives are required for a clocks, and once again, since we’re looking for a
deed to be truly “good.” generally-stated proposition based on the clocks
(B) While it’s claimed that magnanimous behavior example, any choice containing only a clocks-related
helps the immune system, we can hardly say that lack soundbite must be wrong.
of magnanimity actually causes most serious illnesses. (E) mistakenly takes a specific attribute of clocks, the
(C) is way off. Magnanimity produces the chemicals; fact that they increase synchronization and
not the other way around. productivity, and attributes it to most machines. The
(E)’s a distor tion — the stimulus said only that actual functioning of the clock, and the specific effects
magnanimity will stimulate the activity of white blood it has (like adding synchronization), cannot be
cells, so we can’t infer that it will produce new ones. generalized and applied to other machines, which may,
after all, have entirely different functions and effects.
5. (C)
7. (B)
The author concludes that if production costs for
operas were lowered, then operas would no longer The author of the argument has erroneously combined
need corporate sponsorship, but instead could be the premises in the first two sentences and taken them
privately financed; as a result, ticket buyers would be to mean that a person must practice three hours a day to
able to see a wider variety of operas, instead of just the become an expert. This is a case of confusing necessity
most famous ones. (C) destroys this wishful thinking. with sufficiency. True, according to the second sentence,
If, without corporate support, opera companies could three hours a day of practice will make someone an
still afford to produce only the most famous operas, expert eventually (i.e., it’s sufficient for expertise), but
the argument falls apart. nothing rules out the possibility of becoming an expert by
practicing less than three hours a day (i.e., three hours
(A) subtly strengthens the argument, by opening up the
a day is not necessary). According to the first sentence,
possibility that people would be willing to pay to see
at least some practice is necessary, but just because
little known operas as opposed to famous ones.
three hours a day will ensure expertise doesn’t mean
(B) The argument isn’t affected in the least if that someone couldn’t acquire expert status with only
corporate sponsors still wish to support opera, the one hour of practice a day.
whole issue centers on what will happen without
(A) We’re told that a person who practices three hours
corporate sponsorship.
a day will eventually become an expert—it therefore
(D) The author’s whole point is to produce the operas doesn’t matter if someone who practices three hours a
without corporate sponsorship, so this piece of info day is not yet considered an expert.
has no bearing on the argument.(E) We can infer that
(C) is the contrapositive of the conclusion, and therefore
the author has already thought of this, which is why her
makes the same error that the conclusion makes, that
plan involves cutting production costs in order to
of assuming that one must practice three hours a day
produce operas without corporate support.
to become an expert.
6. (A) (D) The argument doesn’t specify that the practice
hours must be consecutive, so the fact that some
The sentence “at the same time that the clock opened
music teachers may frown on this notion is irrelevant.
up some avenues, it closed others” gives it away; it
introduces the notion of something having both a (E) also brings up an irrelevant consideration—the
liberating and a restricting effect. Since we’re looking argument focuses the practice requirements that will
for a proposition (a generality) illustrated by this lead to expertise. How many or how few people have
example, (A) fits the bill by expanding the author’s this kind of time on their hands is outside the scope.

27
PrepTest 3 Explained

8. (E) other people in their presence (what choice (E)


First, define the unexpected finding. There’s discusses) is never discussed.
“incontestable proof” that safety seats will reduce the (D) We have no idea who “most people” are, or what
number of serious injuries sustained by children in car they would prefer.
accidents, yet a large number of children who are riding
in safety seats continue to receive the serious injuries 10. (E)
that the seats were specifically designed to prevent, This one’s fairly complex, but your best bet is to follow
and have been proven to be effective in preventing. If, the cause-and-effect relationships and make
as (E) says, safety seats must be used properly in deductions as if they represented Logic Games rules.
order for them to afford protection, and if parents are Only one of the choices can’t be deuced from the
failing to use them properly, then it’s easily conceivable “rules” of this stimulus, and it turns out to be (E): We
for children riding in safety seats to still suffer serious know that if there’s a decrease in atmospheric carbon,
injuries. there’s a decrease in atmospheric heat. We cannot
(A) and (C) are wrong for the same reason: Both infer the opposite: If there’s a decrease in atmospheric
supposed explanations ignore the fact that the heat, there may or may not be a decrease in
unexpected finding centers around children who are atmospheric carbon.
actually in the seats receiving injuries. (A), (D) Straight from the passage: A decrease in
(B) More children making automobile trips doesn’t atmospheric carbon leads to a decrease in atmosphere
bring us any closer to understanding the mysterious heat, which results in decreased evaporation of sea
failure of the seats to perform the function for which water. The decreased evaporation of sea water results
they were designed, and for which they have been in less rain, which means less carbon being washed
proven effective. into the seas.
(D) This one also explains nothing—it ignores the fact (B) If the amount of carbon in the atmosphere
that children are sustaining the very injuries that the increases, then the atmosphere holds more heat. This
seats were designed to prevent. increased heat in turn results in increased evaporation
of sea water. Put ‘em together, and we get (B):
9. (A) increased carbon in the atmosphere means increased
Since both the willingness to make fun of oneself and evaporation of sea water.
the willingness to allow others to do so are evidence of (C) More of the same process: Increased heat leads to
a self-confident person, we can infer that people who increased evaporation of sea water, which leads to
aren’t self-confident aren’t likely to enjoy being made increased rainfall.
fun of, either by themselves or by others. (A) implies
that no one who lacks self-confidence will enjoy being 11. (E)
made fun of, which may at first seem like a stronger The author concludes that environmentalists are wrong
statement than the argument can definitely support. to worry about increased carbon levels due to the
However, the phrase “the surest mark” is meant to burning of fossil fuels, because nature will continually
indicate that telling funny stories or jokes about adjust the carbon level. However, if, as choice (E)
oneself is tantamount to being self-confident; in other states, the adjustment process works very slowly,
words, if you tell funny stories or jokes about yourself, allowing wide short-term fluctuations in the carbon
then you are self-confident. (A) is the contrapositive of level, then it’s conceivable that dangerous or even
this. Even if you don’t see eye to eye with this logic, as lethal levels of carbon can build up, thus severely
some may not, (A) is still the best of the bunch, and weakening the author’s argument: the
very much in line with the gist of the argument. environmentalists would be right to worry.
(B) Beyond the scope—the passage talks about other (A), (C) The author doesn’t debate the necessity of
people making fun of self-confident people, but says carbon, she merely argues that nature takes care of the
nothing about the reverse scenario. carbon level by itself, and that the environmentalists
(C) and (E) are also beyond the scope. There’s no way should chill.
for us to infer the reason behind the stated correlation (B) Some may feel that this weakens the author’s
(C); that is, why self-confident people put themselves concession in the last sentence, but even if it did, that
down. And the reason why anyone tells jokes about sentence is basically a tangential point that has little to

28
Section IV: Logical Reasoning

do with the logic of the argument, which deals with 13. (C)
nature’s regulation of the carbon level. However, this The inference comes right out of the first sentence: If
probably doesn’t even affect the author’s concession, the only way a bookstore can profitably sell books at
because that statement never said or implied that the below market prices is to get the books at a discount
threat to humans of increased carbon levels would from publishers, then it certainly must be true that a
come from breathing it in; it would likely be more bookstore that is profitably selling books at below
indirect, along the lines of the processes described. market prices is getting discounts from publishers.
(D) This is an irrelevant comparison that has no (A) and (B) both confuse necessity with sufficiency:
bearing on the logic of the argument. For (A), it’s necessary to receive discounts to sell at
below-market prices, but it’s not a guarantee. As for
12. (E) (B), a high volume relies on either exclusive access or
Based on the statistical evidence provided, the author a large specialized market (a necessary condition), but
concludes that if American children are to become as neither of these things necessarily guarantees (i.e., is
capable as their South Korean peers, they must watch sufficient) a high sales volume.
less television. The author doesn’t consider the (D) The bookstore in this choice (one that doesn’t sell
possibility that there are other reasons for this books at below-market prices) is entirely outside of the
difference in mathematical ability. Surely how much scope of the argument, so nothing can be inferred
television children watch isn’t the only difference about it.
between lifestyles of South Korean and American
(E) No; it’s quite possible that a bookstore with
children. In making this claim, therefore, the author
exclusive access to a large specialized market that
assumes that other possible factors that could account
also caters to mass tastes will be able to sell books at
for the difference in math abilities don’t in fact play a
a discount.
role. (E) zeros in on this assumption: it certainly would
seem that mathematical instruction would also be an
14. (D)
important factor in determining competence in math;
therefore, in concluding that less TV watching will help Because this store doesn’t cater to mass tastes, if it
improve U.S. childrens’ math abilities, the author must does not have exclusive access, then it is impossible
assume that math instruction in America and South for that store to generate the volume to get the
Korea is of the same caliber. discounts that would allow the store to profitably sell
its books at below-market prices.
(A) and (B), if true, both weaken the argument by
providing (and not discounting, as (E) does) other (A) could be true; all the store needs is exclusive
explanations: lack of interest and discipline on the part access to a large specialized market.
of American children compared with the South (B) can literally always be true; nothing forces
Koreans. If these things are true, then the impact of TV bookstores to sell at below-market prices, after all.
watching on the situation may be at best insignificant (C) is certainly true. Since this store doesn’t cater to
and at worst irrelevant. mass tastes, then either it has exclusive access to a
(C) The author’s conclusion, simply put, is based on a large specialized market, or it can kiss the volume, and
necessary condition that underlies U.S. children’s by extension, the discounts, goodbye.
math success. The argument ends there; it stands (E) Since this store doesn’t cater to mass tastes, it’s
regardless of whether children will be motivated to certainly possible that the store doesn’t have exclusive
adopt this condition. access or the publisher’s discount.
(D) The conclusion is stated in terms of watching
“less” TV; how much less is not addressed. This one 15. (E)
hour time frame appears out of nowhere, and the Species have been coming and going long before
author need not assume this for the conclusion to humans ever came along, and those who wish to blame
stand. Remember, watching less TV is a necessary recent extinctions on human technology and its
condition, according to the author, but is not sufficient consequent effects on the environment ignore the fact
for math success, so the author need not assume that that extinction is a natural process that would be going
watching less than one hour of TV a day will guarantee on even if we were not around. In other words, the
an increase in math ability. author argues that the more recent extinctions are just
part of the same process that has been going on since

29
PrepTest 3 Explained

before the environment was harmed by technology. 17. (D)


However, she has failed to present any proof that the You should be able to pre-phrase a pretty close answer.
more recently extinct species would have definitely If no risky projects are decided upon in bureaucracies,
become extinct without our help. and the bureaucratic decision making involves “many
(A) is mired in a scope shift—the author doesn’t argue people,” then the author is likely assuming that
that technology has not harmed the environment in any decisions to undertake risky projects are only made by
way; all she argues is that damage to the environment a single individual.
didn’t cause extinctions which wouldn’t have otherwise (A) Try the Denial Test: What if not all projects in a
occurred naturally. bureaucracy involve risk? Does the argument suffer?
(B) The author isn’t ignoring this fact; it’s just not No; the conclusion is about the fate of risky projects,
important to her argument. These species aren’t but it doesn’t rely on the notion that all projects in a
mentioned because they have no effect on her bureaucracy involve risk—some may not without
argument, not because she erroneously overlooks affecting the logic of the argument.
them. (B) What “type” of people work in bureaucracies is
(C) Once again, failing to consider something that has beyond the scope, and certainly not a crucial factor in
nothing to do with your argument is not a reasoning making this argument.
error. The existence of undiscovered species has no (C) contains a few scope shifts: First,taking risks may
bearing on the argument that extinction is natural. be similar but is not necessarily the same thing as
(D) is a distortion of the author’s second statement. under taking risky projects. Second, the choice
She doesn’t identify a group of scientists that have this concerns an individual with decision making power,
theory, she just says that scientists, in general, whereas the stimulus indirectly refers to a situation
estimate that it’s the case. where a single individual has all of the power. If you’re
unsure, try the Denial Test again: It’s possible for an
16. (A) individual with decision-making power to not take risks,
If the public distrusts the media, believing that it might and for the conclusion to still remain valid.
be prejudiced, but they use information dispersed by (E) says that people take fewer risks when they’re
the media as fuel for that doubt, then it seems members of a group. But we need a statement that
probable that the public would find it difficult to detect implies that they take no risks when they’re part of a
a wide-spread media bias. In other words, it would be bureaucracy. So while (E) may have initially appeared
difficult to find an objective standard against which to tempting, it’s not strong or specific enough to allow the
judge a media report.(A) is a rephrasing of that argument’s conclusion to follow from the premises.
thinking.
(B) The public may believe that the media is biased, 18. (E)
and the media may indeed be biased, but there’s no Three things are required in order to achieve the
evidence in the stimulus that there is a specific physicalists’ goal: 1) a knowledge of the basic functions
political agenda at work. of neurons; 2) a knowledge of how neurons interact;
(C) The author hasn’t even established that biases and 3) a delineation of the psychological faculties to be
actually exist, just that the public may have trouble explained. At present, two of these things have already
discerning media bias, if it even exists. The reasons for been attained—we have knowledge of the basic
the bias therefore cannot be a conclusion the author’s functions of neurons, and we understand the scope and
moving towards. character of many psychological capacities. Therefore,
the author concludes, we can expect to achieve the goal
(D) We’re only interested in the bias of the media and
of explaining mental functions in neurobiological terms
the fact that the public would have a hard time seeing
in the near future. What happened to “a knowledge of
it. Whether or not reporters hold the same view as their
how neurons interact?” The flaw is that three things are
public is immaterial. There’s also a scope shift here:
required to achieve a certain result, and the author
The “views” discussed in (D) don’t necessarily fall
concludes that the result can be achieved even though
under the category of “biases,” which is after all what
we only have two.
the stimulus talks about.
(A) The conclusion agrees with, not contradicts, the
(E) We’re given no clues as to how the public would
physicalists.
respond to a scenario like this.

30
Section IV: Logical Reasoning

(B) The author should not be expected to describe 20. (C)


“exactly what is currently known about the basic The politician and his opponents—those who want
functions of neurons.” We’re concerned only that that more funding for low-income housing—agree that
knowledge is needed for the result, and that it currently homelessness is a serious social problem. They
exists. The argument’s validity doesn’t rely on a disagree about how to solve it. Don’t worry yourself
delineation of the specific mechanics involved. with the obvious flaw in his argument. We aren’t asked
(C)’s complaint, that the word “neurobiological” is used that. Focus on the phrase “homelessness is a serious
as if it had the same meaning as “mental”, is the very social problem;” it’s simply information that serves to
opposite of the truth. If the two words had been treated set up the argument, but actually does little more.
as meaning the same thing from the beginning, the Understanding this allows should help you eliminate
argument would have been pointless. The point is that choices that mistakenly ascribe special significance to
ultimately all mental functions will be explainable in this statement, when in fact it’s little more than filler
neurobiological terms. material. And since the argument itself is based on
(D) Outside the scope; the argument isn’t concerned other pieces of evidence, this intro statement is
with whether explaining mental functions in consistent with an acceptance or denial of the
neurobiological terms is useful, only that it will be conclusion.
accomplished in the near future. (A) The statement is merely offered to frame the
argument; it’s not an alternative perspective to the one
19. (D) the politician adopts.
The author argues that the government can win back (B) The politician doesn’t set out to solve the problem,
the lost administrators by raising government salaries he sets out to refute a proposed solution.
to a level that is comparable with private sector (D) The politician admits that homelessness is a
salaries. However, this is possible only if these serious problem; he’s certainly not trying to discredit
administrators will willingly change jobs again, and go this position.
back to the government posts they left behind. If not,
(E) It isn’t necessary for the politician’s conclusion.
the argument falls apart.
His argument that the problem of homelessness can’t
(A) The individuals in question are already experienced, be solved by providing more housing works just as well
and the author feels that it is already worth the extra if homelessness is a minor problem or no problem at all.
money required for the government to “recapture”
them, so any new experience is not necessary to the 21. (B)
worth of this plan.
The reason that Thomas finds it incredible that such a
(B) We learn in the first sentence that these small dietary change could have such a drastic effect
administrators are both experienced and extremely on population is because he’s overlooked the fact that
capable, and both of these qualities are important. overconsumption of eggs isn’t the only cause of death.
Competence can be the most important factor, as When Leona says that 5,000 lives might be saved, she
opposed to experience, in determining how well means that people who would otherwise have died
government agencies function, and the argument from an overconsumption of eggs would be spared.
would still be unaffected, which shows that this (B) is Who’s to say that some of those people wouldn’t die
not a necessary assumption on the part of the author. for some other reason? Therefore, an adequate
(C) Try the Denial Test: The denial of (C) would be the response from Leona would focus on the fact that the
government taking no action and the gap in salaries numbers cited by her represent the number of people
not increasing. This state of affairs does nothing to who without the diet may have died, and with the diet
damage the author’s proposal, which is about how to wouldn’t die, from eating too many eggs; however, she
recapture administrators who have already left. never said that they wouldn’t die for some other
(E) is the opposite of what is assumed. The argument reason.
has said all along that the administrators are chasing (A) and (D) discuss population growth (a major scope
the big money. (E) would have them moving in large shift), which neither clarifies Leona’s claim nor
numbers to the lower-paying (public) government jobs. addresses Thomas’s point. Neither person is claiming
the overall population would grow by 5,000 people per
year; Leona simply argues that 5,000 people who

31
PrepTest 3 Explained

would have died from eating too many eggs would not stolen, besides the allegedly successful program
die for this reason. (there’s that “alternative explanation” concept popping
(C) is irrelevant, as it addresses what would happen if up again). (A) does this by asking if the car owners that
egg consumption were cut by more than half, not by are likely to join the program owners take any other
half, which is the issue. Anyway, if Leona responded special precautions to prevent theft. If the answer to
with choice (C), she would only confuse poor Thomas this question is yes, then perhaps the other measures,
even more, because then his 50,000 figure, which he not the program, are keeping the number of thefts
thought was too high to begin with, would have to be down, which would signify that the stem’s conclusion
even higher. that the program has reduced thefts is unwarranted.
(E) makes no sense as a response to Thomas. The (B), (E) How many neighborhoods the program is
issue isn’t what individual consumers must do to operating in (A) and whether neighborhoods in which
comply with this dietary change—it’s what will be the the program took effect were a representative cross-
result of the dietary change. section of all neighborhoods, in terms of car types (E),
are both irrelevant. The conclusion only claims that the
22. (B) program was successful in the neighborhoods where it
was actually applied.
The key notion in this one is that only after that transfer
occurs can the new therapies help patients. The (C) The program only takes effect between 1 A.M. and
inference turns out to be pretty straightforward. If the 5 A.M., so what happens during the daytime can have
new therapies can help patients only after they have nothing to do with the program. Moreover, the issue
been transferred from the lab to the marketplace, then here is the cars that weren’t stolen, and the reasons
they can’t possibly help patients before that. why they weren’t stolen.
(A) We don’t know that the FDA necessarily regulates (D) While this certainly might be of some concern to
all therapeutic agents after they’ve been put on the the owners, it doesn’t say much about the
market. We’re only told that the FDA regulates the effectiveness of the program. True, if the answer is
actual introduction of these agents. yes, it would indicate that the program is indeed
functioning, but doesn’t help us to evaluate its
(C) All we’re told is that the research community
effectiveness as well as (A) does, the answer of whih
carries out a long process of discovery and testing. For
could blow the conclusion out of the water.
all we know, though, it’s the FDA that is responsible for
this—perhaps the agency requires the long testing
24. (E)
period.
Break the principle down to its essentials: An action is
(D) Maintaining the quality of therapeutic agents is a
morally good only if it 1) benefits another person and
new subject altogether, as is the issue of what the FDA
2) was intended to benefit that other person. An action
should, or shouldn’t, do.
is morally bad if it harms another person, and either 1)
(E) Here’s the necessary/sufficient thing again: Only such harm was intended or 2) a reasonable person
after a new drug has been introduced can it help should have known that harm was likely to occur. The
patients. That is not to say that it necessarily will help only way to find the situation that matches the spirit of
patients. this principle is to work through the choices, and
unfortunately, the testmakers buried the answer in
23. (A) choice (E). Jonathan’s act of neglecting his three-year-
The question stem introduces a conclusion that could old niece caused harm to her, so it meets the first
lead from this stimulus; that automobile theft has been criterion for a morally bad act. Although he intended no
reduced by the program. We’re asked for the choice harm, he should have realized that his failure to watch
that asks the question whose answer is most his niece carefully was likely to lead to harm; (E)’s
important in evaluating that conclusion—that is, whose judgment therefore conforms to the principle in the
answer is most likely to tell us whether or not the stimulus.
conclusion is justified. Since the conclusion in the (A) According to the stimulus, an action is only morally
stem is based on the fact that cars bearing the special bad if it actually causes harm—which the action in this
decals have a lower theft rate than other cars, we can case did not.
expect that the correct question will ask if there might
not be another reason that these cars aren’t being

32
Section IV: Logical Reasoning

(B) Jeffrey’s action indirectly—and that’s the key


word—helped Sara. Since his action was performed
with the intention of securing his own promotion, not
with the intention of benefiting Sara, it fails the
stimulus’ test for a morally good action.
(C) Once again, we have an action that fails the
“morally good” test on the grounds that it doesn’t
benefit anyone (quite the opposite, no less).
(D) The homeless man was obviously harmed, which
puts Marilees’ action in the morally bad ballpark. But it
still needs to pass another test—the harm must be
inflicted intentionally, or the harmful result must be
foreseeable. Neither of these are met here (what are
the chances that someone will choke on a sandwich; it
was most likely the man’s own fault for talking and
chewing at the same time), so the judgment that
Marilees performed a morally bad action doesn’t
conform to the principle in the stimulus.

33
1-800-KAP-TEST | kaptest.com

ÖLL3107Ayä
LL3107A

*LSAT is a registered trademark of the Law School Admission Council. Printed in USA ©2008 Kaplan, Inc.

You might also like